aboutsummaryrefslogtreecommitdiff
diff options
context:
space:
mode:
authorAdrian Kummerlaender2017-02-12 20:03:44 +0100
committerAdrian Kummerlaender2017-02-12 20:03:44 +0100
commitd9ca730a23d404d40e806baae7822e9da0f7e6db (patch)
treeebf5a9035b8a23d45ca4027c6ffd20559caa5f5c
parente7087f26043192217e07a82f3855f68ac54b3e5a (diff)
downloadmath_reference_sheets-d9ca730a23d404d40e806baae7822e9da0f7e6db.tar
math_reference_sheets-d9ca730a23d404d40e806baae7822e9da0f7e6db.tar.gz
math_reference_sheets-d9ca730a23d404d40e806baae7822e9da0f7e6db.tar.bz2
math_reference_sheets-d9ca730a23d404d40e806baae7822e9da0f7e6db.tar.lz
math_reference_sheets-d9ca730a23d404d40e806baae7822e9da0f7e6db.tar.xz
math_reference_sheets-d9ca730a23d404d40e806baae7822e9da0f7e6db.tar.zst
math_reference_sheets-d9ca730a23d404d40e806baae7822e9da0f7e6db.zip
Introduce field aliases
-rw-r--r--analysis.tex178
-rw-r--r--analysis_3.tex38
-rw-r--r--lineare_algebra.tex44
-rw-r--r--numerik_1.tex24
-rw-r--r--zusammenfassung.tex4
5 files changed, 146 insertions, 142 deletions
diff --git a/analysis.tex b/analysis.tex
index 65a98e9..f2e3edb 100644
--- a/analysis.tex
+++ b/analysis.tex
@@ -1,10 +1,10 @@
\section*{Folgen}
-$\displaystyle\lim_{n \to \infty} = a \Leftrightarrow \forall \epsilon > 0 \exists N_\epsilon \in \mathbb{N} \forall n \geq N_\epsilon : | a_n - a | \leq \epsilon$
+$\displaystyle\lim_{n \to \infty} = a \Leftrightarrow \forall \epsilon > 0 \exists N_\epsilon \in \N \forall n \geq N_\epsilon : | a_n - a | \leq \epsilon$
\subsection*{Konvergenzsatz für monotone Folgen}
-Sei $(a_n)$ wachsend und nach oben beschränkt \\ $\Rightarrow \exists \lim_{n \to \infty} a_n = \sup_{n\geq 1} a_n := \sup\{a_n | n \in \mathbb{N}\}$
+Sei $(a_n)$ wachsend und nach oben beschränkt \\ $\Rightarrow \exists \lim_{n \to \infty} a_n = \sup_{n\geq 1} a_n := \sup\{a_n | n \in \N\}$
Analoges für fallende, nach unten beschr. Folgen.
@@ -18,15 +18,15 @@ $\underline\lim_{x \to \infty} a_n$ Minimum der Häufungspunkte
\subsection*{Cauchyfolgen}
-$\forall \epsilon > 0 \exists N_\epsilon \in \mathbb{N} \forall n, m \geq N_\epsilon : | a_n - a_m | \leq \epsilon$
+$\forall \epsilon > 0 \exists N_\epsilon \in \N \forall n, m \geq N_\epsilon : | a_n - a_m | \leq \epsilon$
Cauchyfolge $\Leftrightarrow$ konvergente Folge
\subsection*{Uneigentliche Grenzwerte}
-Sei $\overline{\mathbb{R}} = \mathbb{R} \cup \{-\infty, +\infty\}$
+Sei $\overline{\R} = \R \cup \{-\infty, +\infty\}$
-$\forall K \in \mathbb{N} \exists N_K \in \mathbb{N} \forall n \geq N_K : x_n \geq K \Leftrightarrow \displaystyle\lim_{n \to \infty} x_n = \infty$
+$\forall K \in \N \exists N_K \in \N \forall n \geq N_K : x_n \geq K \Leftrightarrow \displaystyle\lim_{n \to \infty} x_n = \infty$
\subsection*{Beispiele und Hinweise}
@@ -46,20 +46,20 @@ $\sum_{k\geq 0} a_k$ konvergiert $\Rightarrow (a_k)$ ist Nullfolge.
\subsubsection*{Leibnizkriterium}
-Es gelte $\forall k \in \mathbb{N}_0 : b_k \geq b_{k+1} \geq 0$ und $\displaystyle \lim_{k \to \infty} b_k = 0$
+Es gelte $\forall k \in \N_0 : b_k \geq b_{k+1} \geq 0$ und $\displaystyle \lim_{k \to \infty} b_k = 0$
Dann konvergiert: $\sum_{k=0}^\infty (-1)^k b_k$
\subsubsection*{Majorantenkriterium}
\begin{enumerate}[label=(\alph*)]
- \item Wenn $0 \leq |a_k| \leq b_k \forall k \in \mathbb{N}_0$ und $\sum_k b_k$ konvergiert, dann konvergiert $\sum_k a_k$ absolut und es gilt $|\sum_{k=0}^\infty a_k| \leq \sum_{k=0}^\infty |a_k| \leq \sum_{k=0}^\infty b_k$.
+ \item Wenn $0 \leq |a_k| \leq b_k \forall k \in \N_0$ und $\sum_k b_k$ konvergiert, dann konvergiert $\sum_k a_k$ absolut und es gilt $|\sum_{k=0}^\infty a_k| \leq \sum_{k=0}^\infty |a_k| \leq \sum_{k=0}^\infty b_k$.
\item Wenn $a_k \geq b_k \geq 0$ und $\sum_k b_k$ divergiert, dann divergiert $\sum_k a_k$
\end{enumerate}
\subsubsection*{Quotientenkriterium}
-Sei $(a_n)_{n\geq 0}$ Folge, $n_0 \in \mathbb{N}$ mit $\forall n \geq n_0 : a_n \neq 0$
+Sei $(a_n)_{n\geq 0}$ Folge, $n_0 \in \N$ mit $\forall n \geq n_0 : a_n \neq 0$
$\overline\lim_{n \to \infty} |\frac{a_{n+1}}{a_n}| < 1 \implies \sum a_n$ konvergiert absolut
@@ -96,7 +96,7 @@ Die allgemeine harmonische Reihe $\sum_{k=0}^\infty \frac{1}{k^\alpha}$ divergie
Eine Reihe $\sum_k a_k$ konvergiert genau dann, wenn:
-$\forall \epsilon > 0 \exists N_\epsilon \in \mathbb{N} \forall n > m \geq N_\epsilon : | \sum_{k=m+1}^{n} a_k | \leq \epsilon$
+$\forall \epsilon > 0 \exists N_\epsilon \in \N \forall n > m \geq N_\epsilon : | \sum_{k=m+1}^{n} a_k | \leq \epsilon$
\section*{Potenzreihen}
@@ -148,21 +148,21 @@ Wenn $z_0 \in D$ ein isolierter Punkt, dann ist $f$ in $z_0$ immer stetig.
\subsection*{Abgeschlossenheit}
-$D \subseteq \mathbb{C}$ ist abgeschlossen, wenn $z_n \in D \text{ für } n \in \mathbb{N} \land \lim_{n \to \infty} z_n = z \implies z \in D$.
+$D \subseteq \mathbb{C}$ ist abgeschlossen, wenn $z_n \in D \text{ für } n \in \N \land \lim_{n \to \infty} z_n = z \implies z \in D$.
\subsection*{Gleichmäßige Stetigkeit}
-Sei $D \subseteq \mathbb{R}, f : D \rightarrow \mathbb{R}$. $f$ ist glm. stetig gdw.:
+Sei $D \subseteq \R, f : D \rightarrow \R$. $f$ ist glm. stetig gdw.:
$\forall \epsilon > 0 \exists \delta > 0 \forall x, x_0 \in D : | x - x_0 | < \delta \\ \hspace*{4mm} \implies | f(x) - f(x_0) | < \epsilon$
\subsection*{Satz vom Maximum}
-Sei $D \subseteq \mathbb{C}$ abgeschlossen und beschränkt, $f: D \rightarrow \mathbb{R}$ stetig, dann nimmt $f$ auf $D$ ein Maximum und Minimum an, ist also insbesondere beschränkt.
+Sei $D \subseteq \mathbb{C}$ abgeschlossen und beschränkt, $f: D \rightarrow \R$ stetig, dann nimmt $f$ auf $D$ ein Maximum und Minimum an, ist also insbesondere beschränkt.
\subsection*{Zwischenwertsatz}
-Sei $f: [a, b] \rightarrow \mathbb{R}$ stetig, dann:
+Sei $f: [a, b] \rightarrow \R$ stetig, dann:
$f([a, b]) = [\min_{[a, b]} f, \max_{[a, b]} f]$.
@@ -172,13 +172,13 @@ $\forall y \in [\min_{[a, b]} f, \max_{[a, b]} f] \exists x \in [a, b]: f(x)=y$
\subsection*{Nullstellensatz}
-Sei $f \in [a, b] \rightarrow \mathbb{R}$ stetig mit $f(a)f(b) \leq 0$, dann:
+Sei $f \in [a, b] \rightarrow \R$ stetig mit $f(a)f(b) \leq 0$, dann:
$\exists x \in [a, b]: f(x) = 0$
\subsection*{Intervallsatz}
-Sei $I \subseteq \mathbb{R}$ ein Intervall und $f : I \rightarrow \mathbb{R}$ stetig.
+Sei $I \subseteq \R$ ein Intervall und $f : I \rightarrow \R$ stetig.
Dann ist $f(I)$ ein Intervall.
@@ -188,7 +188,7 @@ Dann ist $f(I)$ ein Intervall.
Fkt. Folge $(f_n)$ konv. punktweise gegen $f$, wenn:
-$\forall z \in D \forall \epsilon > 0 \exists N_{\epsilon, z} \in \mathbb{N} \forall n \geq N_{\epsilon, z} : | f_n(z) - f(z) | \leq \epsilon$
+$\forall z \in D \forall \epsilon > 0 \exists N_{\epsilon, z} \in \N \forall n \geq N_{\epsilon, z} : | f_n(z) - f(z) | \leq \epsilon$
Dies ist äquivalent zu $\lim_{n\to \infty} f_n(x)=f(x)$.
@@ -196,7 +196,7 @@ Dies ist äquivalent zu $\lim_{n\to \infty} f_n(x)=f(x)$.
Fkt. Folge $(f_n)$ konv. gleichmäßig gegen $f$, wenn:
-$\forall z \in D \exists N_\epsilon \in \mathbb{N} \forall n \geq N_\epsilon : \sup_{z \in D} | f_n(z) - f(z) | \leq \epsilon$
+$\forall z \in D \exists N_\epsilon \in \N \forall n \geq N_\epsilon : \sup_{z \in D} | f_n(z) - f(z) | \leq \epsilon$
Dies ist äquivalent zu $\displaystyle\lim_{n\to \infty}(\sup_{z \in D}| f_n(z) - f(z) |) = 0$.
@@ -204,7 +204,7 @@ Dies ist äquivalent zu $\displaystyle\lim_{n\to \infty}(\sup_{z \in D}| f_n(z)
\subsection*{Differenzierbarkeit}
-Funktion $f : I \rightarrow \mathbb{R}$ ist in $x_0$ differenzierbar, wenn:
+Funktion $f : I \rightarrow \R$ ist in $x_0$ differenzierbar, wenn:
$\lim_{x \to x_0} \frac{f(x) - f(x_0)}{x - x_0} =: f'(x_0) = \frac{df}{dx}(x_0)$ existiert.
@@ -245,7 +245,7 @@ $\lim_{x \to x_0} \frac{f(x) - f(x_0)}{x - x_0} =: f'(x_0) = \frac{df}{dx}(x_0)$
\subsection*{Mittelwertsatz}
-Sei $a < b$ in $\mathbb{R}$ und $f, g \in C^1((a, b),\mathbb{R})$, dann:
+Sei $a < b$ in $\R$ und $f, g \in C^1((a, b),\R)$, dann:
$\exists \xi \in (a, b) : ( f(b) - f(a) ) g'(\xi) = f'(\xi)(g(b) - g(a))$
@@ -273,20 +273,20 @@ Seien $x, y > 0$, $p \in (1, \infty)$ und $p' := \frac{p}{p-1}$, also $\frac{1}{
\subsection*{L'Hospital}
-Sei $-\infty \leq a < b \leq +\infty$, $f, g : (a, b) \rightarrow \mathbb{R}$ differenzierbar mit $\forall x \in (a, b) : g'(x) \neq 0$ und es gelte eines:
+Sei $-\infty \leq a < b \leq +\infty$, $f, g : (a, b) \rightarrow \R$ differenzierbar mit $\forall x \in (a, b) : g'(x) \neq 0$ und es gelte eines:
\begin{enumerate}[label=(\alph*)]
\item $\exists \lim_{x \to b^-} f(x) = \lim_{x \to b^-} g(x) = 0$
\item $\exists \lim_{x \to b^-} g(x) = \pm \infty$
\end{enumerate}
-Ferner existiere $l \in \overline{\mathbb{R}}$ mit $\lim_{x \to b^-} \frac{f'(x)}{g'(x)} = l$.
+Ferner existiere $l \in \overline{\R}$ mit $\lim_{x \to b^-} \frac{f'(x)}{g'(x)} = l$.
Dann existiert $\lim_{x \to b^-} \frac{f(x)}{g(x)} = l$.
\subsection*{Umkehrregel}
-Sei $f : I \subseteq \mathbb{R} \to \mathbb{R}$ strikt monoton, stetig und differenzierbar in $x_0 \in I$ mit $f'(x_0) \neq 0$. Dann ist $f^{-1} : f(I) \to \mathbb{R}$ in $y_0 = f(x_0)$ differenzierbar mit:
+Sei $f : I \subseteq \R \to \R$ strikt monoton, stetig und differenzierbar in $x_0 \in I$ mit $f'(x_0) \neq 0$. Dann ist $f^{-1} : f(I) \to \R$ in $y_0 = f(x_0)$ differenzierbar mit:
$(f^{-1})'(y_0) = \frac{1}{f'(f^{-1}(y_0))} = \frac{1}{f'(x_0)}$
@@ -308,7 +308,7 @@ $R_{n-1, x_0} f(x) = \frac{1}{n!} f^{(n)} (x_0 + \theta(x-x_0))(x-x_0)^n$
\subsection*{Integralrestglied}
-Sei $f \in C^{n+1}((a, b)), n \in \mathbb{N}_0, x_0 \in (a, b)$, dann gilt:
+Sei $f \in C^{n+1}((a, b)), n \in \N_0, x_0 \in (a, b)$, dann gilt:
$f(x) - T_{n,x_0} f(x) = \frac{1}{n!} \int_{x_0}^x (x-t)^n f^{(n+1)}(t) dt$
@@ -316,7 +316,7 @@ $f(x) - T_{n,x_0} f(x) = \frac{1}{n!} \int_{x_0}^x (x-t)^n f^{(n+1)}(t) dt$
\subsection*{Stückweise Stetigkeit}
-$f : [a, b] \rightarrow \mathbb{R}$ ist stückweise stetig, wenn sich $[a, b]$ in endlich viele disjunkte Teilintervalle so aufteilen lässt, dass $f$ eingeschränkt auf diese stetig ist.
+$f : [a, b] \rightarrow \R$ ist stückweise stetig, wenn sich $[a, b]$ in endlich viele disjunkte Teilintervalle so aufteilen lässt, dass $f$ eingeschränkt auf diese stetig ist.
$PC([a, b])$ ist Menge stckw. stg. Fkt. über $[a, b]$.
@@ -340,13 +340,13 @@ $\int_a^b f(\phi(x))\phi'(x) dx = \int_{\phi(a)}^{\phi(b)} f(y) dy$
\subsection*{Uneigentliche Riemann-Integrale}
-Sei $-\infty < a < b \leq +\infty, f : [a, b) \rightarrow \mathbb{R}$ so, dass $\forall \beta \in (a, b): f|_{[a, \beta]} \in PC([a, \beta])$.
+Sei $-\infty < a < b \leq +\infty, f : [a, b) \rightarrow \R$ so, dass $\forall \beta \in (a, b): f|_{[a, \beta]} \in PC([a, \beta])$.
-Falls $\lim_{\beta \to b^-} \int_a^\beta f(x) dx =: \int_a^b f(x) dx$ in $\mathbb{R}$ existiert, heißt $f$ uneigentlich Riemann-integrierbar.
+Falls $\lim_{\beta \to b^-} \int_a^\beta f(x) dx =: \int_a^b f(x) dx$ in $\R$ existiert, heißt $f$ uneigentlich Riemann-integrierbar.
\subsubsection*{Majoranten- / Minorantenkriterium}
-Sei $-\infty < a < b \leq +\infty$ und $f, g : [a, b) \to \mathbb{R}$ mit $\forall c \in (a, b) : f, g \in PC([a, c])$, dann:
+Sei $-\infty < a < b \leq +\infty$ und $f, g : [a, b) \to \R$ mit $\forall c \in (a, b) : f, g \in PC([a, c])$, dann:
\begin{enumerate}[label=(\alph*)]
\item $\forall x \in [a, b) : |f(x)| \leq g(x) \land g$ uneigentlich integrierbar $\Rightarrow$ $f, |f|$ uneigentlich integrierbar.
@@ -388,7 +388,7 @@ Sei $-\infty < a < b \leq +\infty$ und $f, g : [a, b) \to \mathbb{R}$ mit $\fora
Folge $(x_n) \subseteq X$ konvergiert gegen $x \in X$, wenn:
-$\forall \epsilon > 0 \exists N_\epsilon \in \mathbb{N} \forall n \geq N_\epsilon : ||x_n - x|| \leq \epsilon$
+$\forall \epsilon > 0 \exists N_\epsilon \in \N \forall n \geq N_\epsilon : ||x_n - x|| \leq \epsilon$
\subsubsection*{$p$-Norm}
@@ -399,13 +399,13 @@ $$|x|_p := \begin{cases}
\subsubsection*{Hölder-Ungleichung}
-Sei $p \in |1, \infty]$ mit $\frac{1}{p}+\frac{1}{p'}=1$ sowie $x, y \in \mathbb{K}^m$:
+Sei $p \in |1, \infty]$ mit $\frac{1}{p}+\frac{1}{p'}=1$ sowie $x, y \in \K^m$:
$|\sum_{k=1}^m x_k y_k| \leq \sum_{k=1}^m |x_k y_k| \leq |x|_p |x|_{p'}$
\subsubsection*{Minkowski-Ungleichung}
-Sei $x, y \in \mathbb{K}^m$: $|x+y|_p \leq |x|_p + |y|_p$
+Sei $x, y \in \K^m$: $|x+y|_p \leq |x|_p + |y|_p$
\subsubsection*{Cauchy-Schwarz-Ungleichung}
@@ -427,7 +427,7 @@ $||\cdot|| \sim |||\cdot||| \Leftrightarrow \exists C, c > 0 \forall x \in X \la
\subsubsection*{Konvergenz bezüglich $p$-Normen}
-Sei $1 \leq p < q \leq \infty$, $x \in \mathbb{K}^m$, dann gilt:
+Sei $1 \leq p < q \leq \infty$, $x \in \K^m$, dann gilt:
$|x|_p \leq m^{\frac{1}{p} - \frac{1}{q}} |x|_q$ und $|x|_q \leq |x|_p$
@@ -435,7 +435,7 @@ $|x|_p \leq m^{\frac{1}{p} - \frac{1}{q}} |x|_q$ und $|x|_q \leq |x|_p$
Sei $(X, ||\cdot||)$ normierter Vektorraum. $(x_n) \subseteq X$ ist Cauchyfolge, wenn gilt:
-$\forall \epsilon > 0 \exists N_\epsilon \in \mathbb{N} \forall n, m \geq N_\epsilon : ||x_n - x_m|| \leq \epsilon$
+$\forall \epsilon > 0 \exists N_\epsilon \in \N \forall n, m \geq N_\epsilon : ||x_n - x_m|| \leq \epsilon$
\subsection*{Normen stetiger Funktionen}
@@ -454,9 +454,9 @@ Wenn jede Cauchyfolge in $(X, ||\cdot||)$ einen Grenzwert besitzt, dann heißt $
\subsubsection*{Hilbertraum}
-Die Norm des Banachraums $(\mathbb{K}^m, |\cdot|_2)$ ist durch Skalarprodukt gegeben. Ein solcher Banachraum heißt Hilbertraum.
+Die Norm des Banachraums $(\K^m, |\cdot|_2)$ ist durch Skalarprodukt gegeben. Ein solcher Banachraum heißt Hilbertraum.
-Ein $\mathbb{R}$-VRaum mit SKP ist ein euklidischer Raum.
+Ein $\R$-VRaum mit SKP ist ein euklidischer Raum.
\subsubsection*{Äquivalenz der Normen}
@@ -474,13 +474,13 @@ Sei $V$ endlichdim. VRaum, dann sind alle Normen auf $V$ äquivalent. Insb. ist
$(x_n) \subseteq X$ konvergiert in $(M, d)$ gegen $x \in X$, wenn:
-$\forall \epsilon > 0 \exists N_\epsilon \in \mathbb{N} \forall n \geq N_\epsilon : d(x_n, x) \leq \epsilon$
+$\forall \epsilon > 0 \exists N_\epsilon \in \N \forall n \geq N_\epsilon : d(x_n, x) \leq \epsilon$
\subsubsection*{Cauchyfolgen bzgl. Metriken}
Eine Folge $(x_n) \subseteq M$ heißt Cauchfolge, wenn:
-$\forall \epsilon > 0 \exists N_\epsilon \in \mathbb{N} \forall n, m \geq N_\epsilon : d(x_n, x_m) \leq \epsilon$
+$\forall \epsilon > 0 \exists N_\epsilon \in \N \forall n, m \geq N_\epsilon : d(x_n, x_m) \leq \epsilon$
Ein metrischer Raum ist vollständig, wenn jede Cauchfolge in $(M, d)$ konvergiert.
@@ -560,7 +560,7 @@ $\forall x, y \in M : d_N(f(x), f(y)) \leq L d_M(x, y)$
Lipschitz stetige Fkt. sind insb. auch glm. stetig.
-$L(V, W) = \{T : V \rightarrow W | T \text{ linear} \}$ sind lineare Abb. zwischen $\mathbb{K}$-VR $V$ und $W$, genannt Operatoren.
+$L(V, W) = \{T : V \rightarrow W | T \text{ linear} \}$ sind lineare Abb. zwischen $\K$-VR $V$ und $W$, genannt Operatoren.
\subsubsection*{Operatornorm}
@@ -594,7 +594,7 @@ $f$ bijektiv $\Rightarrow f$ ist homömorph, d.h. bijektiv, stetig und hat steti
\subsubsection*{Satz vom Maximum}
-Sei $K$ komp. metr. Raum, $f: K \rightarrow \mathbb{R}$ stetig, dann:
+Sei $K$ komp. metr. Raum, $f: K \rightarrow \R$ stetig, dann:
$\exists x_{\pm} \in K : f(x_+) = \displaystyle\max_{x \in K} f(x) \land f(x_-) = \displaystyle\min_{x \in K} f(x)$
@@ -606,19 +606,19 @@ $\forall x, y \in M \exists w \in C([0,1], M) : w(0) = x \land w(1) = y$
\subsubsection*{Zwischenwertsatz}
-Seien $M$, $N$ metr. Räume, $M$ wegzusammenhängend und $f \in C(M, N)$. Dann ist $f(M)$ wegzusammenhängend. Für $N = \mathbb{R}$ ist $f(M)$ ein Intervall.
+Seien $M$, $N$ metr. Räume, $M$ wegzusammenhängend und $f \in C(M, N)$. Dann ist $f(M)$ wegzusammenhängend. Für $N = \R$ ist $f(M)$ ein Intervall.
\section*{Differentialrechnung in VRäumen}
\subsection*{Kurventangente}
-Sei $J$ ein Intervall. Ein Weg ist stetige Abbildung $f = (f_1 \hdots f_m)^T : J \rightarrow \mathbb{R}^m$. Das Bild $\Gamma = f(J)$ heißt Kurve. $f$ ist Parametrisierung von $\Gamma$.
+Sei $J$ ein Intervall. Ein Weg ist stetige Abbildung $f = (f_1 \hdots f_m)^T : J \rightarrow \R^m$. Das Bild $\Gamma = f(J)$ heißt Kurve. $f$ ist Parametrisierung von $\Gamma$.
Die Ableitung eines $C^1$-Wegs $f$ in $t_0 \in J$ ist def.:
-$f'(t_0) = \displaystyle\lim_{h \to 0} \frac{1}{h} (f(t_0 + h) - f(t_0)) = \begin{pmatrix} f_1'(t_0) \\ \vdots \\ f_m'(t_0)\end{pmatrix} \in \mathbb{R}^m$
+$f'(t_0) = \displaystyle\lim_{h \to 0} \frac{1}{h} (f(t_0 + h) - f(t_0)) = \begin{pmatrix} f_1'(t_0) \\ \vdots \\ f_m'(t_0)\end{pmatrix} \in \R^m$
-Für $f'(t_0) \neq 0$ ist Tangente $T(\mathbb{R})$ an $f(t_0)$ durch $T(t) = f(t_0) + (t - t_0)f'(t_0)$ mit $t \in \mathbb{R}$ gegeben.
+Für $f'(t_0) \neq 0$ ist Tangente $T(\R)$ an $f(t_0)$ durch $T(t) = f(t_0) + (t - t_0)f'(t_0)$ mit $t \in \R$ gegeben.
\subsection*{Partielle Ableitungen}
@@ -634,31 +634,31 @@ $f'(x_0) := A$ ist Ableitung von $f$ bei $x_0$. Wenn $\forall x_0 \in D : f $ di
\subsubsection*{Jacobimatrix}
-Sei $D \subseteq \mathbb{R}^l$ offen, $f : D \rightarrow \mathbb{R}^k$, $i \in \{1, ..., k\}$, $j \in \{1, ..., l\}$ und $f = (f_1 \hdots f_m)^T$, dann:
+Sei $D \subseteq \R^l$ offen, $f : D \rightarrow \R^k$, $i \in \{1, ..., k\}$, $j \in \{1, ..., l\}$ und $f = (f_1 \hdots f_m)^T$, dann:
$\partial f(x) = \begin{pmatrix} \partial_1 f_1(x) & \hdots & \partial_l f_1(x) \\ \vdots & & \vdots \\ \partial_1 f_k(x) & \hdots & \partial_l f_k(x) \end{pmatrix}$
\subsubsection*{Gradient}
-Wenn $D \subseteq \mathbb{R}^m$ offen und $f : D \rightarrow \mathbb{R}$ bei $x \in D$ differenzierbar ist, dann:
+Wenn $D \subseteq \R^m$ offen und $f : D \rightarrow \R$ bei $x \in D$ differenzierbar ist, dann:
-$\nabla f(x) := \begin{pmatrix} \partial_1 f(x) \\ \vdots \\ \partial_m f(x) \end{pmatrix} = f'(x)^T \in \mathbb{R}^m$
+$\nabla f(x) := \begin{pmatrix} \partial_1 f(x) \\ \vdots \\ \partial_m f(x) \end{pmatrix} = f'(x)^T \in \R^m$
Identifiziert kritische Stellen mit $\nabla f(x) = 0$.
\subsection*{Richtungsableitung}
-Sei $D \subseteq \mathbb{R}^m$ offen, $f : D \rightarrow \mathbb{R}$ und $v \in \mathbb{R}^m \setminus \{0\}$, dann ist Ableitung von $f$ bei $x$ in Richtung $v$:
+Sei $D \subseteq \R^m$ offen, $f : D \rightarrow \R$ und $v \in \R^m \setminus \{0\}$, dann ist Ableitung von $f$ bei $x$ in Richtung $v$:
$\partial_v f(x) = \frac{\partial f}{\partial v}(x) := \displaystyle\lim_{t \to 0} \frac{1}{t} (f(x+tv)-f(x))$
-Insofern Grenzwert in $\mathbb{R}$ existiert. Weiterhin gilt:
+Insofern Grenzwert in $\R$ existiert. Weiterhin gilt:
-$\partial_v f(x) = (\nabla f(x) | v)$ für $f \in C^1(D, \mathbb{R})$.
+$\partial_v f(x) = (\nabla f(x) | v)$ für $f \in C^1(D, \R)$.
\subsubsection*{Hessematrix}
-Wenn $D \subseteq \mathbb{R}^m$ offen und $f \in C^n(D, \mathbb{R})$, dann $\nabla f \in C^1(D,\mathbb{R}^m)$ und somit:
+Wenn $D \subseteq \R^m$ offen und $f \in C^n(D, \R)$, dann $\nabla f \in C^1(D,\R^m)$ und somit:
$\nabla^2 f(x) = \begin{pmatrix} \partial_1 \partial_1 f(x) & \hdots & \partial_m \partial_1 f(x) \\ \vdots & & \vdots \\ \partial_1 \partial_m f(x) & \hdots & \partial_m \partial_m f(x) \end{pmatrix}$
@@ -666,7 +666,7 @@ $\nabla^2 f(x) = \begin{pmatrix} \partial_1 \partial_1 f(x) & \hdots & \partial_
\subsection*{Taylor in mehreren Dimensionen}
-Sei $D \subseteq \mathbb{R}^l$ offen, $f \in C^{n+1}(D, \mathbb{R})$, $x \in D$, $r > 0$ mit $\overline B(x,r) \subseteq D$ und $h \in \mathbb{R}^l$ mit $|h|_2 < r$, dann:
+Sei $D \subseteq \R^l$ offen, $f \in C^{n+1}(D, \R)$, $x \in D$, $r > 0$ mit $\overline B(x,r) \subseteq D$ und $h \in \R^l$ mit $|h|_2 < r$, dann:
$T_{n,x} f(x+h) = f(x) + \displaystyle\sum_{j=1}^n \frac{1}{j!} \displaystyle\sum_{\alpha_1, ..., \alpha_j=1}^l h_{\alpha_1} \cdot \hdots \cdot h_{\alpha_j} ( \partial_{\alpha_1} \hdots \partial_{\alpha_j} f )(x)$
@@ -680,18 +680,18 @@ $R_{n,x} f(x+h) = f(x+h) - T_{n,x}f(x+h)$
\subsection*{Definitheit}
-Sei $A \in L(\mathbb{R}^m)$ symmetrisch, dann ist $A$:
+Sei $A \in L(\R^m)$ symmetrisch, dann ist $A$:
\begin{description}[leftmargin=!,labelwidth=28mm]
- \item[positiv definit] $\forall v \in \mathbb{R}^m \setminus \{0\} : (Av|v) > 0$
- \item[negativ definit] $\forall v \in \mathbb{R}^m \setminus \{0\} : (Av|v) > 0$
- \item[positiv semidefinit] $\forall v \in \mathbb{R}^m : (Av|v) \geq 0$
- \item[negativ semidefinit] $\forall v \in \mathbb{R}^m : (Av|v) \leq 0$
+ \item[positiv definit] $\forall v \in \R^m \setminus \{0\} : (Av|v) > 0$
+ \item[negativ definit] $\forall v \in \R^m \setminus \{0\} : (Av|v) > 0$
+ \item[positiv semidefinit] $\forall v \in \R^m : (Av|v) \geq 0$
+ \item[negativ semidefinit] $\forall v \in \R^m : (Av|v) \leq 0$
\end{description}
\subsubsection*{Definitheit von $2\times 2$ Hessematrizen}
-Sei $A = \begin{pmatrix} a & b \\ b & d \end{pmatrix} \in \mathbb{R}^{2 \times 2}$ eine symmetrische Matrix:
+Sei $A = \begin{pmatrix} a & b \\ b & d \end{pmatrix} \in \R^{2 \times 2}$ eine symmetrische Matrix:
\begin{enumerate}[label=(\alph*)]
\item positiv definit $\Leftrightarrow$ $a > 0, ad > b^2$
@@ -712,7 +712,7 @@ Sei $A = \begin{pmatrix} a & b \\ b & d \end{pmatrix} \in \mathbb{R}^{2 \times 2
\subsection*{Extremstellen}
-Seien $D \subseteq \mathbb{R}^m$ offen, $f \in C^2(D, \mathbb{R})$, $x \in D$, dann:
+Seien $D \subseteq \R^m$ offen, $f \in C^2(D, \R)$, $x \in D$, dann:
\begin{description}[leftmargin=!,labelwidth=28mm]
\item[Maximum] $\nabla^2 f(x)$ negativ semidefinit
@@ -728,26 +728,26 @@ Wenn $\nabla f(x) = 0$:
\subsection*{Umkehrsatz}
-Seien $D \subseteq \mathbb{R}^m$ offen, $f \in C^1(D, \mathbb{R}^m)$, $x_0 \in D$, $y_0 = f(x_0)$, $f'(x_0) \in L(\mathbb{R}^m$ bijektiv, dann:
+Seien $D \subseteq \R^m$ offen, $f \in C^1(D, \R^m)$, $x_0 \in D$, $y_0 = f(x_0)$, $f'(x_0) \in L(\R^m$ bijektiv, dann:
-$\exists U \subseteq D \text{ offen }, V \subseteq \mathbb{R}^m : x_0 \in U, y_0 \in V, f_U : U \rightarrow V \text{ bijektiv }, V \subseteq f(D), (f_U)^{-1} \in C^1(V, U), \forall x \in U : f'(x) \text{ invertierbar}$. Insbesondere:
+$\exists U \subseteq D \text{ offen }, V \subseteq \R^m : x_0 \in U, y_0 \in V, f_U : U \rightarrow V \text{ bijektiv }, V \subseteq f(D), (f_U)^{-1} \in C^1(V, U), \forall x \in U : f'(x) \text{ invertierbar}$. Insbesondere:
$\forall y=f(x) \in V : (f_U^{-1})'(y) = f'(f_U^{-1}(y))^{-1} = f'(x)^{-1}$
\subsubsection*{Diffeomorphismen}
-Seien $D \subseteq \mathbb{R}^m$ offen, $f \in C^1(D, \mathbb{R}^m)$, $\tilde D = f(D)$, $f$ injektiv, $\forall x \in D: f'(x)$ invertierbar. Dann ist $\tilde D$ offen und $f : D \rightarrow \tilde D$ ein Diffeomorphismus, d.h. $D$, $\tilde D$ offen, $f$ bijektiv und $f \in C^1(D, \mathbb{R}^m)$, $f^{-1} \in C^1(\tilde D, \mathbb{R}^m)$
+Seien $D \subseteq \R^m$ offen, $f \in C^1(D, \R^m)$, $\tilde D = f(D)$, $f$ injektiv, $\forall x \in D: f'(x)$ invertierbar. Dann ist $\tilde D$ offen und $f : D \rightarrow \tilde D$ ein Diffeomorphismus, d.h. $D$, $\tilde D$ offen, $f$ bijektiv und $f \in C^1(D, \R^m)$, $f^{-1} \in C^1(\tilde D, \R^m)$
\subsubsection*{Polarkoordinaten}
-$\phi : D = (\mathbb{R} \setminus \{0\}) \times \mathbb{R} \rightarrow \mathbb{R}^2; (r, \varphi) \mapsto \begin{pmatrix} r \cos \varphi \\ r \sin \varphi \end{pmatrix} = \begin{pmatrix} x \\ y \end{pmatrix}$
+$\phi : D = (\R \setminus \{0\}) \times \R \rightarrow \R^2; (r, \varphi) \mapsto \begin{pmatrix} r \cos \varphi \\ r \sin \varphi \end{pmatrix} = \begin{pmatrix} x \\ y \end{pmatrix}$
\subsection*{Satz über implizit definierte Funktionen}
-Seien $D \subseteq \mathbb{R}^{m+k}$ offen, $f \in C^1(D, \mathbb{R}^k)$, $(x_0, y_0) \in D$ mit $f(x_0, y_0) = 0$ und $(\partial_y f)(x_0, y_0) \in L(\mathbb{R}^k)$ bijektiv. Dann $\exists \text{ offene } U_x \subseteq \mathbb{R}^m, U_y \subseteq \mathbb{R}^k$ sowie Abbildung $\varphi \in C^1(U, \mathbb{R}^k)$ mit:
+Seien $D \subseteq \R^{m+k}$ offen, $f \in C^1(D, \R^k)$, $(x_0, y_0) \in D$ mit $f(x_0, y_0) = 0$ und $(\partial_y f)(x_0, y_0) \in L(\R^k)$ bijektiv. Dann $\exists \text{ offene } U_x \subseteq \R^m, U_y \subseteq \R^k$ sowie Abbildung $\varphi \in C^1(U, \R^k)$ mit:
\begin{enumerate}[label=(\alph*)]
- \item $(x_0, y_0) \in U_x \times U_y \subseteq D$, $\varphi(U_x) \subseteq U_y$, $\varphi(x_0)=y_0$, $\forall (x, y) \in U_x \times U_y : \partial_y f(x,y) \in L(\mathbb{R}^k)$ bijektiv
+ \item $(x_0, y_0) \in U_x \times U_y \subseteq D$, $\varphi(U_x) \subseteq U_y$, $\varphi(x_0)=y_0$, $\forall (x, y) \in U_x \times U_y : \partial_y f(x,y) \in L(\R^k)$ bijektiv
\item $(x, y) \in U_x \times U_y \land f(x,y)=0 \\ \Leftrightarrow x \in U_x \land y=\varphi(x)$
\item $\varphi'(x) = -[(\partial_y f)(x, \varphi(x))]^{-1}(\partial_x f)(x, \varphi(x))$
\end{enumerate}
@@ -756,32 +756,32 @@ Insbesondere auch $\forall x \in U_x : f(x, \varphi(x))=0$
\subsection*{$C^1$-Flächen}
-$M \subseteq \mathbb{R}^3$ ist eingebettete $C^1$-Fläche, wenn $\forall p \in M \exists \text{ offene } V, U \subseteq \mathbb{R}^3 : p \in V \land \psi : V \rightarrow U$ so, dass $\psi(V \cap M) = U \cap (\mathbb{R}^2 \times \{0\})$. $\psi$ heißt dann Karte.
+$M \subseteq \R^3$ ist eingebettete $C^1$-Fläche, wenn $\forall p \in M \exists \text{ offene } V, U \subseteq \R^3 : p \in V \land \psi : V \rightarrow U$ so, dass $\psi(V \cap M) = U \cap (\R^2 \times \{0\})$. $\psi$ heißt dann Karte.
\subsubsection*{Charakterisierung}
\begin{enumerate}[label=(\alph*)]
\item $M$ ist $C^1$-Fläche
- \item $\forall p \in M \exists \text{ offenes } D \subseteq \mathbb{R}^3 \land g \in C^1(D, \mathbb{R}) : \\ p \in D \land \forall w \in D : \nabla g(w) \neq 0 \land M \cap D = \{w\in D | g(w) = 0\}$ (lokale Nullstellenmenge)
- \item $\forall p \in M \exists i < j \in \{1, 2, 3\}, \text{ offene } U_1 \subseteq \mathbb{R}^1, U_2 \subseteq \mathbb{R}^2, h \in C^1(U_2, U_1) : p_k \in U_1, (p_i, p_j) \in U_2$ mit $\{k\} = \{1, 2, 3\} \setminus \{j, j\}$, $h(U_2) \subseteq U_1$ und für $Z=\{x \in \mathbb{R}^3 | (x_i, x_j) \in U_2, x_k \in U_1\}$ gilt $M \cap Z = \{x \in \mathbb{R}^3 | (x_i, x_j) \in U_2, x_k = h(x_i, x_j)\}$ (lokaler Graph)
- \item $\forall p \in M \exists \text{ offenes } U_0 \subseteq \mathbb{R}^2, W \subseteq \mathbb{R}^3, F \in C^1(U_0, \mathbb{R}^3) : p \in W, \forall (s, t) \in U_0 : Rg(F'(s, t)) = 2$ und $F: U_0 \rightarrow M \cap W$ bijektiv mit stetiger Umkehrabbildung. (lokale Parameterisierung)
+ \item $\forall p \in M \exists \text{ offenes } D \subseteq \R^3 \land g \in C^1(D, \R) : \\ p \in D \land \forall w \in D : \nabla g(w) \neq 0 \land M \cap D = \{w\in D | g(w) = 0\}$ (lokale Nullstellenmenge)
+ \item $\forall p \in M \exists i < j \in \{1, 2, 3\}, \text{ offene } U_1 \subseteq \R^1, U_2 \subseteq \R^2, h \in C^1(U_2, U_1) : p_k \in U_1, (p_i, p_j) \in U_2$ mit $\{k\} = \{1, 2, 3\} \setminus \{j, j\}$, $h(U_2) \subseteq U_1$ und für $Z=\{x \in \R^3 | (x_i, x_j) \in U_2, x_k \in U_1\}$ gilt $M \cap Z = \{x \in \R^3 | (x_i, x_j) \in U_2, x_k = h(x_i, x_j)\}$ (lokaler Graph)
+ \item $\forall p \in M \exists \text{ offenes } U_0 \subseteq \R^2, W \subseteq \R^3, F \in C^1(U_0, \R^3) : p \in W, \forall (s, t) \in U_0 : Rg(F'(s, t)) = 2$ und $F: U_0 \rightarrow M \cap W$ bijektiv mit stetiger Umkehrabbildung. (lokale Parameterisierung)
\end{enumerate}
\subsubsection*{Tangentialraum}
Tangentialraum an $m$ bei $p=F(u_0)$ wobei $F$ lokale Parametrisierung:
-$T_p M = F'(u_0)(\mathbb{R}^2) = \left\{ F'(u_0) \begin{pmatrix} s \\ t \end{pmatrix} \middle| s, t \in \mathbb{R} \right\}$
+$T_p M = F'(u_0)(\R^2) = \left\{ F'(u_0) \begin{pmatrix} s \\ t \end{pmatrix} \middle| s, t \in \R \right\}$
\subsubsection*{Normalenraum}
Normalenraum an $M$ bei $p$ und $g(p)=0$ wobei $g$ lokale Nullstellenmenge:
-$N_p M = lin\{\nabla g(p)\} = \{t\nabla g(p) | t \in \mathbb{R}\}$
+$N_p M = lin\{\nabla g(p)\} = \{t\nabla g(p) | t \in \R\}$
\subsection*{Lagrange}
-Seien $D \subseteq \mathbb{R}^l$ offen, $f \in C^1(D, \mathbb{R}$ und $g \in C^1(D, \mathbb{R}^k)$ mit $1 \leq k \leq l$. Setze $M=\{z\in D| g(z)=0\}$. $f$ besitze auf $M$ ein Extremum bei $z_0 \in M$ und $g'(z_0)$ habe Rang $k$. Dann existieren Lagrangesche Multiplikatoren $\lambda_1, ..., \lambda_k \in \mathbb{R}$ so, dass:
+Seien $D \subseteq \R^l$ offen, $f \in C^1(D, \R$ und $g \in C^1(D, \R^k)$ mit $1 \leq k \leq l$. Setze $M=\{z\in D| g(z)=0\}$. $f$ besitze auf $M$ ein Extremum bei $z_0 \in M$ und $g'(z_0)$ habe Rang $k$. Dann existieren Lagrangesche Multiplikatoren $\lambda_1, ..., \lambda_k \in \R$ so, dass:
$\nabla f(z_0) = \lambda_1\nabla g_1(z_0) + \hdots + \lambda_k\nabla g_k(z_0)$
@@ -789,13 +789,13 @@ Ferner gilt $g_1(z_0) = 0, \hdots, g_k(z_0)=0$.
\section*{Kurvenintegrale}
-Sei $\gamma \in C([a, b], \mathbb{R}^m)$ ein Weg und $\Gamma = \gamma([a, b])$ die dazugehörige Kurve. Die Länge von $\Gamma$ wird durch Polygonzüge approximiert. Sei dazu $a \leq b$ und $\mathcal{Z}(a, b)$ die Menge aller Zerlegungen $Z = \{a = t_0, t_1, ..., t_{n-1}, t_n = b\}$. Für $Z \in \mathcal{Z}$ wird gesetzt:
+Sei $\gamma \in C([a, b], \R^m)$ ein Weg und $\Gamma = \gamma([a, b])$ die dazugehörige Kurve. Die Länge von $\Gamma$ wird durch Polygonzüge approximiert. Sei dazu $a \leq b$ und $\mathcal{Z}(a, b)$ die Menge aller Zerlegungen $Z = \{a = t_0, t_1, ..., t_{n-1}, t_n = b\}$. Für $Z \in \mathcal{Z}$ wird gesetzt:
$l(\gamma, Z) = \sum_{j=1}^n |\gamma(t_j) - \gamma(t_{j-1})|_2$
\subsection*{Rektifizierbarkeit}
-$\gamma : [a, b] \rightarrow \mathbb{R}^m$ ist rektifizierbar, wenn:
+$\gamma : [a, b] \rightarrow \R^m$ ist rektifizierbar, wenn:
$l_{[a, b]}(\gamma) = ||\gamma||_{BV} := \sup_{Z\in \mathcal{Z}} l(\gamma, Z) < \infty$
@@ -803,35 +803,35 @@ $l(\gamma)$ ist dann die Länge von $\gamma$.
\subsubsection*{Wegintegral}
-Sei $\gamma \in C^1([a, b], \mathbb{R}^m)$, dann ist $\gamma$ rektifizierbar mit:
+Sei $\gamma \in C^1([a, b], \R^m)$, dann ist $\gamma$ rektifizierbar mit:
$l(\gamma) = \int_a^b |\gamma'(t)|_2 dt$
\subsection*{Kurvenintegrale und Potentiale}
-Sei $\gamma \in C([a, b], \mathbb{R}^m)$ stückweise $C^1$, $\Gamma = \gamma([a, b])$.
+Sei $\gamma \in C([a, b], \R^m)$ stückweise $C^1$, $\Gamma = \gamma([a, b])$.
\subsubsection*{Kurvenintegral erster Art}
-Sei reelles $f \in C(\Gamma, \mathbb{R})$ gegeben:
+Sei reelles $f \in C(\Gamma, \R)$ gegeben:
\vspace*{-5mm}
$$\int_\Gamma f d\gamma = \int_\Gamma f(x) d\gamma := \int_a^b f(\gamma(t)) | \gamma'(t) |_2 dt$$
\subsubsection*{Kurvenintegral zweiter Art}
-Sei vektorwertiges $F \in C(\Gamma, \mathbb{R}^m)$ gegeben:
+Sei vektorwertiges $F \in C(\Gamma, \R^m)$ gegeben:
\vspace*{-5mm}
$$\int_\Gamma F \cdot dx = \int_\Gamma F(x) \cdot dx := \int_a^b (F(\gamma(t))|\gamma'(t)) dt$$
\subsubsection*{Wegunabhängigkeit}
-Sei $D \subseteq \mathbb{R}^m$ offen, dann ist $F \in C(D, \mathbb{R}^m)$ wegunabhängig auf $D$, wenn für alle stückweisen $C^1$-Kurven $\gamma_1, \gamma_2 \in C([a, b], \mathbb{R}^m)$ in $D$ mit gleichem Anfangs- und Endpunkt gilt:
+Sei $D \subseteq \R^m$ offen, dann ist $F \in C(D, \R^m)$ wegunabhängig auf $D$, wenn für alle stückweisen $C^1$-Kurven $\gamma_1, \gamma_2 \in C([a, b], \R^m)$ in $D$ mit gleichem Anfangs- und Endpunkt gilt:
$$\int_{\Gamma_1} F \cdot dx = \int_{\Gamma_2} F \cdot dx$$
-Ein $\phi \in C^1(D, \mathbb{R})$ heißt Potential von $F$ auf $D$, wenn $\nabla\phi = F$ auf $D$. $F$ ist dann Gradientenfeld.
+Ein $\phi \in C^1(D, \R)$ heißt Potential von $F$ auf $D$, wenn $\nabla\phi = F$ auf $D$. $F$ ist dann Gradientenfeld.
Weiterhin ist $F$ wegunabhängig auf $D$ gdw. $F$ ein Potential $\phi$ auf $D$ hat.
@@ -839,13 +839,13 @@ Ferner gilt dann: $\int_\Gamma F \cdot dx = \phi(\gamma(b))-\phi(\gamma(a))$
\subsubsection*{Poincar\'e}
-Sei $D \subseteq \mathbb{R}^m$ offen und sternförmig und $F \in C^1(D, \mathbb{R}^m)$ sei rektifizierbar. Dann hat $F$ ein Potential auf $D$, insbesondere auch auf jeder Kugel $B(x_0, r) \subseteq D$.
+Sei $D \subseteq \R^m$ offen und sternförmig und $F \in C^1(D, \R^m)$ sei rektifizierbar. Dann hat $F$ ein Potential auf $D$, insbesondere auch auf jeder Kugel $B(x_0, r) \subseteq D$.
\section*{Gewöhnliche Differentialgleichung}
\subsection*{Lokale Lipschitzstetigkeit}
-Sei $M \subseteq \mathbb{R}^m$ und $J$ ein Intervall. $g : J \times M \rightarrow \mathbb{R}^k$ ist lokal Lipschitz in $x$, wenn:
+Sei $M \subseteq \R^m$ und $J$ ein Intervall. $g : J \times M \rightarrow \R^k$ ist lokal Lipschitz in $x$, wenn:
$\forall (t_0, x_0) \in J \times M \exists \delta = \delta(t_0, x_0) > 0, r = r(t_0, x_0) > 0, L = L(t_0, x_0) \geq 0 \forall t \in [t_0 - \delta, t_0 + \delta] \cap J \forall x, y \in \overline B(x_0, r) \cap M : |g(t,x) - g(t, y)|_2 \leq L|x-y|_2$.
@@ -857,7 +857,7 @@ Ein diff. $f$ ist Lipschitz gdw. $\partial_x^{(1)} f$ beschränkt ist.
\subsection*{Anfangswertprobleme}
-Seien $J \subseteq \mathbb{R}$ ein Intervall, $t_0 \in J$ mit $t_0 < \sup J$, $D \subseteq \mathbb{R}^m$ offen, $f \in C(J \times D, \mathbb{R}^m)$ und $u_0 \in D$.
+Seien $J \subseteq \R$ ein Intervall, $t_0 \in J$ mit $t_0 < \sup J$, $D \subseteq \R^m$ offen, $f \in C(J \times D, \R^m)$ und $u_0 \in D$.
\vspace*{-4mm}
\begin{align*}
@@ -865,17 +865,17 @@ Seien $J \subseteq \mathbb{R}$ ein Intervall, $t_0 \in J$ mit $t_0 < \sup J$, $D
u(t_0) &= u_0
\end{align*}
-Für das Anfangswertproblem wird ein $t_1 \in J$ mit $t_1 > t_0$ und eine eindeutige Lösung $u \in C^1([t_0, t_1], \mathbb{R}^m)$ auf $[t_0, t_1]$ gesucht.
+Für das Anfangswertproblem wird ein $t_1 \in J$ mit $t_1 > t_0$ und eine eindeutige Lösung $u \in C^1([t_0, t_1], \R^m)$ auf $[t_0, t_1]$ gesucht.
\subsubsection*{Lokale Lipschitzstetigkeit im Kontext}
-Sei $f \in C(J \times D, \mathbb{R}^k)$, $D \subseteq \mathbb{R}^m$ offen, $J$ ein Intervall und es existieren alle partiellen Ableitungen $\frac{\partial}{\partial x_j} f \in C(J \times D, \mathbb{R}^k)$ für $j \in \{1, \hdots, m\}$.
+Sei $f \in C(J \times D, \R^k)$, $D \subseteq \R^m$ offen, $J$ ein Intervall und es existieren alle partiellen Ableitungen $\frac{\partial}{\partial x_j} f \in C(J \times D, \R^k)$ für $j \in \{1, \hdots, m\}$.
Dann ist $f$ lokal Lipschitz in $x$.
\subsubsection*{Picard-Lindelöf (lokal)}
-Seien $J$ ein Intervall, $D \subseteq \mathbb{R}^m$ offen, $f \in C(J \times D, \mathbb{R}^m)$ lokal Lipschitz in $x$, $u_0 \in D$, $t_0 \in J$ mit $t_0 < \sup J$. Dann gelten:
+Seien $J$ ein Intervall, $D \subseteq \R^m$ offen, $f \in C(J \times D, \R^m)$ lokal Lipschitz in $x$, $u_0 \in D$, $t_0 \in J$ mit $t_0 < \sup J$. Dann gelten:
\begin{enumerate}[label=(\alph*)]
\item $\exists t_1 = t_1(u_0) > t_0 $ mit $t_1 \in J$ und eine eindeutige Lösung $u$ auf $[t_0, t_1]$ von $u'(t) = f(t, u(t))$ mit $u(t_0) = u_0$
@@ -897,11 +897,11 @@ Sei $J$ ein Intervall, $\varphi \in C(J)$, $\varphi \geq 0$, $t_0 = \min J$ und
\subsubsection*{Obergrenze des Existenzintervall}
-Seien $J = [t_0, \infty)$, $D \subseteq \mathbb{R}^m$ offen, $M \subseteq \mathbb{R}^m$ abgeschlossen mit $M \subseteq D$, $f \in C(J \times D, \mathbb{R}^m)$ sei lokal Lipschitz in $x$, $u$ die maximale Lösung von $u'(t) = f(t, u(t))$ auf $[t_0, t(u_0))$ und es gelte $\forall t_0 \leq t \leq \overline t(u_0) : u(t) \in M$. Dann gelten:
+Seien $J = [t_0, \infty)$, $D \subseteq \R^m$ offen, $M \subseteq \R^m$ abgeschlossen mit $M \subseteq D$, $f \in C(J \times D, \R^m)$ sei lokal Lipschitz in $x$, $u$ die maximale Lösung von $u'(t) = f(t, u(t))$ auf $[t_0, t(u_0))$ und es gelte $\forall t_0 \leq t \leq \overline t(u_0) : u(t) \in M$. Dann gelten:
\begin{enumerate}[label=(\alph*)]
\item $\forall b > t_0 \exists c(b) \geq 0 : \forall t \in [t_0, b], x \in M : (f(t,x)|x) \leq c(b) (1+|x|_2^2) \Rightarrow \overline t(u_0) = \infty$
- \item Sei speziell $f: D \rightarrow \mathbb{R}^n$ und $\exists c \geq 0$ mit $\forall x \in M : (f(x)|x) \leq c(1+|x|_2)$, dann $\overline t(u_0) = \infty$
+ \item Sei speziell $f: D \rightarrow \R^n$ und $\exists c \geq 0$ mit $\forall x \in M : (f(x)|x) \leq c(1+|x|_2)$, dann $\overline t(u_0) = \infty$
\end{enumerate}
Bedingung (a) folgt aus: $\forall b > t_0 \exists \tilde c(b) \geq 0 \\ \hspace*{4mm} \forall t \in [t_0, b], x \in M : |f(t,x)|_2 \leq \tilde c(b)(1+|x|_2)$
@@ -912,7 +912,7 @@ Bedingung (b) folgt aus: $\exists \tilde c > 0 \forall x \in M : \\ \hspace*{4mm
Jedes Anfangswertproblem $k$-ter Ordnung lässt sich in ein Problem 1. Ordnung umschreiben.
-Beispielsweise: Das Problem 2. Ordnung $u''(t)=h(t)-u(t)+u'(t)^2$ mit $u(0)=u_0$ und $u'(0)=u_1$ sowie $h \in C(\mathbb{R}, \mathbb{R})$ wird formuliert als Problem 1. Ordnung:
+Beispielsweise: Das Problem 2. Ordnung $u''(t)=h(t)-u(t)+u'(t)^2$ mit $u(0)=u_0$ und $u'(0)=u_1$ sowie $h \in C(\R, \R)$ wird formuliert als Problem 1. Ordnung:
$$\begin{pmatrix}u(t)\\u'(t)\end{pmatrix}' = \begin{pmatrix}u'(t)\\u''(t)=h(t)-u(t)+u'(t)^2\end{pmatrix}$$
@@ -920,7 +920,7 @@ Sei $v_0(t):=u(t)$, $v_1(t):=u'(t)$ und $v(t):=\begin{pmatrix}v_0(t)\\v_1(t)\end
\vspace*{-4mm}
\begin{align*}
- g : \mathbb{R}^3 &\rightarrow \mathbb{R}^2 \\
+ g : \R^3 &\rightarrow \R^2 \\
\begin{pmatrix}t\\v_0\\v_1\end{pmatrix} &\mapsto \begin{pmatrix}v_1\\h(t)-v_0+v_1^2\end{pmatrix}
\end{align*}
@@ -934,7 +934,7 @@ Insgesamt also:
\subsubsection*{Trennung der Variablen}
-Sei $u'(t)=g(t)h(u(t))$ mit $u(t_0)=u_0$ Anfangswertproblem mit $g \in C(\mathbb{R}, \mathbb{R})$, $h \in C((a, b), \mathbb{R})$, $u_0 \in (a, b)$ und $h(u_0) \neq 0$. $u$ ist Lösung, wenn $J$ Intervall mit $\forall t \in J : u(t) \in (a, b)$, $u \in C^1(J, \mathbb{R})$ und $t_0 \in J$.
+Sei $u'(t)=g(t)h(u(t))$ mit $u(t_0)=u_0$ Anfangswertproblem mit $g \in C(\R, \R)$, $h \in C((a, b), \R)$, $u_0 \in (a, b)$ und $h(u_0) \neq 0$. $u$ ist Lösung, wenn $J$ Intervall mit $\forall t \in J : u(t) \in (a, b)$, $u \in C^1(J, \R)$ und $t_0 \in J$.
\vspace*{-5mm}
$$u \text{ ist Lösung } \Rightarrow \int_{t_0}^t g(s) ds = \int_{u_0}^{u(t)} \frac{1}{h(x)} dx$$
diff --git a/analysis_3.tex b/analysis_3.tex
index d524b8d..483a52f 100644
--- a/analysis_3.tex
+++ b/analysis_3.tex
@@ -23,18 +23,18 @@ Ein Mengensystem $\mathcal{A} \subseteq \mathcal{P}(X)$ ist $\sigma$-Algebra auf
\begin{enumerate}[label=(\alph*)]
\item $X \in \mathcal{A}$
\item $A \in \mathcal{A} \Rightarrow A^c := X\setminus A \in \mathcal{A}$
- \item $\forall j \in \mathbb{N} : A_j \in \mathcal{A} \Rightarrow \bigcup_{j\in \mathbb{N}} A_j \in \mathcal{A}$
+ \item $\forall j \in \N : A_j \in \mathcal{A} \Rightarrow \bigcup_{j\in \N} A_j \in \mathcal{A}$
\end{enumerate}
\subsection*{Eigenschaften von $\sigma$-Algebren}
-Seien $\mathcal{A}$ eine $\sigma$-Algebra auf $X$, $n \in \mathbb{N}$, $\forall j \in \mathbb{N} : A_j \in \mathcal{A}$, dann ist $\mathcal{A}$ nach den folgenden Eigenschaften abgeschlossen unter abzählbaren Mengenoperationen:
+Seien $\mathcal{A}$ eine $\sigma$-Algebra auf $X$, $n \in \N$, $\forall j \in \N : A_j \in \mathcal{A}$, dann ist $\mathcal{A}$ nach den folgenden Eigenschaften abgeschlossen unter abzählbaren Mengenoperationen:
\begin{enumerate}[label=(\alph*)]
\item $\emptyset = X^c \in \mathcal{A}$
\item $A_1 \bigcup \cdots \bigcup A_n \in \mathcal{A}$
\item $A_1 \bigcap \cdots \bigcap A_n \in \mathcal{A}$
- \item $\bigcap_{j\in \mathbb{N}} A_j \in \mathcal{A}$
+ \item $\bigcap_{j\in \N} A_j \in \mathcal{A}$
\item $A_1 \setminus A_2 := A_1 \bigcap A_2^c \in \mathcal{A}$
\end{enumerate}
@@ -62,9 +62,9 @@ Sei $\emptyset \neq \mathcal{E} \subseteq \mathcal{P}(X)$, dann gilt:
Sei $X$ ein metrischer Raum und $\mathcal{O}(X)$ das System der in $X$ offenen Mengen, dann ist $\mathcal{B}(X) := \sigma(\mathcal{O}(X))$ die Borelsche $\sigma$-Algebra auf $X$.
-Im Speziellen wird $\mathcal{B}_m := \mathcal{B}(\mathbb{R}^m)$ gesetzt.
+Im Speziellen wird $\mathcal{B}_m := \mathcal{B}(\R^m)$ gesetzt.
-$\mathcal{B}_m$ enthält insb. alle offenen und abgeschlossenen Mengen in $\mathbb{R}^m$ sowie deren abzählbaren Vereinigungen und Durchschnitte.
+$\mathcal{B}_m$ enthält insb. alle offenen und abgeschlossenen Mengen in $\R^m$ sowie deren abzählbaren Vereinigungen und Durchschnitte.
\subsubsection*{Charakterisierung}
@@ -82,7 +82,7 @@ $\mu : \mathcal{A} \rightarrow [0, \infty]$ ist positives Maß auf $\mathcal{A}$
\begin{enumerate}[label=(\alph*)]
\item $\mu(\emptyset) = 0$
- \item $\forall \text{ disjunkte } \{A_j | j \in \mathbb{N}\} \subseteq \mathcal{A} :\\ \hspace*{4mm} \mu(\dot\bigcup_{j\in \mathbb{N}} A_j) = \sum_{j\in \mathbb{N}} \mu(A_j)$
+ \item $\forall \text{ disjunkte } \{A_j | j \in \N\} \subseteq \mathcal{A} :\\ \hspace*{4mm} \mu(\dot\bigcup_{j\in \N} A_j) = \sum_{j\in \N} \mu(A_j)$
\end{enumerate}
\subsection*{Maßraum}
@@ -104,21 +104,21 @@ Dieses wird Punkt- / Diracmaß auf $\mathcal{A}$ genannt.
\subsection*{Zählmaß}
-Sei $\mathcal{A} = \mathcal{P}(\mathbb{N})$ und $\forall j \in \mathbb{N} : p_j \in [0, \infty]$ fest gewählt.
+Sei $\mathcal{A} = \mathcal{P}(\N)$ und $\forall j \in \N : p_j \in [0, \infty]$ fest gewählt.
-$\mu(A) := \sum_{j\in A} p_j$ für $A \subseteq \mathbb{N}$ ist Maß auf $\mathcal{P}(\mathbb{N})$.
+$\mu(A) := \sum_{j\in A} p_j$ für $A \subseteq \N$ ist Maß auf $\mathcal{P}(\N)$.
-Gilt zusätzlich $\forall j \in \mathbb{N} : p_j = 1$ so heißt $\mu$ Zählmaß.
+Gilt zusätzlich $\forall j \in \N : p_j = 1$ so heißt $\mu$ Zählmaß.
\subsection*{Eigenschaften von Maßen}
-Sei $(X, \mathcal{A}, \mu)$ Maßraum und $A, B, A_j \in \mathcal{A}$ für $j \in \mathbb{N}$.
+Sei $(X, \mathcal{A}, \mu)$ Maßraum und $A, B, A_j \in \mathcal{A}$ für $j \in \N$.
\begin{description}[leftmargin=!,labelwidth=26mm]
\item[Monotonie] $A \subseteq B \Rightarrow \mu(A) \leq \mu(B)$
- \item[$\sigma$-Subadditivität] $\mu(\dot\bigcup_{j\in \mathbb{N}} A_j) \leq \sum_{j\in \mathbb{N}} \mu(A_j)$
-